5. Find the quadratic equation with root alpha and beta,given that alpha-beta=2 and alpha^2-beta^2=3.​

Answers

Answer 1

Answer:

[tex]x^{2}-\frac{3}{2} x-\frac{7}{16} =0[/tex]

Step-by-step explanation:

[tex]\begin{cases}\alpha -\beta =2\\ \alpha^{2} -\beta^{2} =3\end{cases}[/tex]

[tex]\Longleftrightarrow \begin{cases}\alpha -\beta =2&\\ \left( \alpha +\beta \right) \left( \alpha -\beta \right) =3&\end{cases}[/tex]

[tex]\Longleftrightarrow \begin{cases}\alpha -\beta =2&\\ \alpha +\beta =\frac{3}{2} &\end{cases}[/tex]

Then

2α = 2 + 3/2 = 7/2

2β = (3/2) - 2 = -1/2

Then

Then

α = 7/4

β = -1/4

Then

a quadratic equation with root α and β can be :

[tex]\left( x+\frac{1}{4} \right) \left( x-\frac{7}{4} \right) =0[/tex]

[tex]\Longrightarrow x^{2}-\frac{3}{2} x-\frac{7}{16} =0[/tex]


Related Questions

One positive number is 8 times another number. Their difference is 70.
Which of the following equations could be used to find the numbers?

Answers

Answer:

X equals 10.

8x - x = 70

Step-by-step explanation:

8 x 10 = 80

80 - 10 = 70

T(h) = - A - B * cos((pi*h)/12) Where T is the temperature in degrees Fahrenheit and h is the number of hours from midnight (0 <= h <= 24) . a. The initial temperature at midnight was - 15 degrees * F and at Noon of New Year's Day it was 5 degrees * F Find A and B. b. Find the average temperature for the first 10 hours. C. Find an expression for the rate that the temperature is changing with respect to h.

Answers

The value of A and B is 5 , 10 ;The average temperature for first ten hours is  -7.466 degree F ; an expression for the rate that the temperature is changing with respect to h ; dt/dh = +B sin ((pi*h)/12)

What is a Function ?

A function is a mathematical statement used to correlate two variables.

The function given is

T(h) = - A - B * cos((pi*h)/12)

T is the temperature in degrees Fahrenheit and

h is the number of hours from midnight (0 <= h <= 24)

a. The initial temperature at midnight was - 15 degrees * F and

at Noon of New Year's Day it was 5 degrees * F

At midnight h = 0

T(h) = -15 degrees F

-15 = -A-B* cos 0

-15 = -A-B

A+B = 15------------------- (1)

At noon , h = 12 , T(h)= 5

- A - B*  cos(pi) = 5

-A - B (-1) = 5

B -A = 5 ----------------------- (2)

Solving equation 1 and 2

2B = 20

B = 10

A = 5

The value of A and B is 5 , 10

The average temperature for first ten hours is  -7.466 degree F

Find an expression for the rate that the temperature is changing with respect to h

dt/dh = +B sin ((pi*h)/12)

To know more about Function

https://brainly.com/question/12431044

#SPJ1

What are m and b in the linear equation y=16+6x

Answers

Answer: B=16 and m=6

Step-by-step explanation:

in y=mx+b the number next to x is always the m/slope and the number without a variable is always the b.

To purchase 13700 worth of restaurant equipment for her business Maria made a down payment of 1500 and took out a business loan for the rest after 3 years of paying monthly payments of 371.16 she finally paid off the loan
What was the total amount Maria ended up paying for the equipment

How much internet did Maria pay on the loan

Answers

The total amount Maria ended up paying for the equipment will be $14,861.76. And The interest of Maria on the loan will be 8.48%.

What is Algebra?

The analysis of mathematical representations is algebra, and the handling of those symbols is logic.

To purchase 13700 worth of restaurant equipment for her business.

Maria made a down payment of 1500 and took out a business loan for the rest, after 3 years of paying monthly payments of 371.16 she finally paid off the loan.

The total amount Maria ended up paying for the equipment will be

Total amount = 371.16 × 3 × 12 + 1500

Total amount = $14,861.76

The interest of Maria on the loan will be

Interset = [(14861.76 – 13700) / 13700] x 100

Interset = 8.48%

More about the Algebra link is given below.

https://brainly.com/question/953809

#SPJ1

A sells an article to B at profit of 20% and B sells it to C at a profit of 25%. If C buys it for Rs.225, what did A pay for it?​

Answers

Answer:

A pay 120 for it

Let the cost price be X

A Sells to B

profit = 20%

= cost price + 20%

= X + 20/100 of X

= 6x/ 5

B sells to C

profit = 25%

= Cost price + + 25%

= cost price + 25%

= 6x/5 + 25/100*6x/5

= 6x/5.+ 3x/10.

= 3x/2

3x/2 = 225.

3x = 450

x = 150.

sp = C. p + profit

S. p - profit = C. p

X - X/5

4x/5

4(150)/5.

120

The Strikers soccer team has 20 members, and 8 of them play offense. What percent of the team members play offense?

Answers

Answer:

40%

Step-by-step explanation:

We already have our first value 20 and the second value 8. Let's assume the unknown value is Y which answer we will find out.

As we have all the required values we need, Now we can put them in a simple mathematical formula as below:

Step 1 ⇒ Y = 8/20

By multiplying both numerator and denominator by 100 we will get:

Step 2 ⇒ Y = 8/20 × 100/100 = 40/100

Step 3 ⇒ Y = 40

Finally, we have found the value of Y which is 40 and that is our answer.

Which of the following terms best fits this definition?

The angle between two sides of a triangle.

Select one:

AAS Theorem


SAS Postulate


Included Side


ASA Postulate


HL Congruence Theorem


Included Angle


SSS Postulate

Answers

Step-by-step explanation:

hope you can understand

Need help with the following question

Answers

3 times. Draw a line 1 unit up, and you can see that there are 3 intersections.

|4x + 7| − 4 = 20
Can anyone help me with this

Answers

Answer:

x = -31/4  or x = 17/4

Step-by-step explanation:

|4x + 7| − 4 = 20

⇔ |4x + 7| − 4 + 4 = 20 + 4

⇔ |4x + 7| = 24

⇔ 4x + 7 = 24  or 4x + 7 = -24

⇔ 4x = 24 - 7 or 4x = -24 - 7

⇔ 4x = 17 or 4x = -31

⇔ x = 17/4 or x = -31/4

Answer:

x = [tex]\frac{17}{4}[/tex]    (17/4 = 4.25)

or x =  [tex]-\frac{31}{4}[/tex]  (-31/4 = -7.75)

Step-by-step explanation:

| | is notation for absolute value

absolute value - the distance that a number is from 0

> essentially, you can think of absolute value as the "positive version" of whatever is inside of the | |

if we have | x | = 20, we could really have (without the | | ) two versions of x

either | x | = 20  ; or | -x | = 20  {because a negative x inside of the | | has the same value as positive x}

we set this up as two equations:

x = 20        or         -x = 20

                                ^ {multiply by -1}

x = 20  or   x = -20

now, let's plug our understanding into the equation

|4x + 7| - 4 = 20

first, we should simplify our equation to:

|4x + 7| - 4 = 20

           + 4   + 4

 |4x + 7|   =   24

now, let's separate this absolute value equation into two separate equations:

4x + 7 = 24:

      - 7    - 7  {subtract 7 from both sides to isolate x}

4x = 17

÷4  ÷4        {divide both sides by 4 to get 1x}

x= [tex]\frac{17}{4}[/tex]

or,

4x + 7 = -24:

      - 7    -7   {subtract 7 from both sides to isolate x}

4x     =    -31

÷4           ÷4   {divide both sides by 4 to get 1x}

 x = [tex]-\frac{31}{4}[/tex]

so, we know that

x = [tex]\frac{17}{4}[/tex]    (17/4 = 4.25)

or x =  [tex]-\frac{31}{4}[/tex]  (-31/4 = -7.75)

hope this helps!!

1
2
3
5
10
Two runners are saving money to attend a marathon.
The first runner has $110 in savings, received a $45 gift
from a friend, and will save $25 each month. The
second runner has $50 in savings and will save $60
each month.
After how many months will both runners have the
same amount of money?
02
O 3

Answers

Answer:

3 months

Step-by-step explanation:

The first runner: $110 + $45 = $155 starting out, plus 25x for $25 each month.

The second runner: $50 starting out, plus 60x for $60 each month.

To find out when both runners have the same amount of money, we will set the expressions equal to each other and solve.

155 + 25x = 60x + 50

105 = 35x

x = 3

Brainliest, please :)

!!!!!!!!!!!!!!!!!!!!!!!!!!!!!!!!!!!!!!!!

Answers

[tex]\quad \huge \quad \quad \boxed{ \tt \:Answer }[/tex]

[tex] \texttt{ \:The absolute maxima of f is f(-8) = 6} [/tex]

____________________________________

[tex] \large \tt Solution \: : [/tex]

Absolute maxima is the maximum possible value for a given x, of a function.

and here, the maximum value is at -8, and the maximum value is 6.

[tex]\qquad \tt \rightarrow \: maximum - \: \: f( - 8) = 6[/tex]

Answered by : ❝ AǫᴜᴀWɪᴢ ❞

Find the value below using your calculator. Round to the nearest hundredth. e³ Find the value below using your calculator . Round to the nearest hundredth . e³​

Answers

The value of e³ = 20.079.

What is Euler's number , e ?

e is an irrational number , It is a numerical constant used in mathematical constant .

The value of e = 2.718

It is asked in the question to determine the value of e³

Therefore

e³ = ( 2.718)³

e³ = 20.079

Therefore the value of e³ = 20.079.

The image of the calculation is attached with the answer.

To know more about Euler's number

https://brainly.com/question/573685

#SPJ1

A woman passed gas silently. I said "it stinks", and she said "I apologize. Excuse me". Why did she say both of those things?

Answers

She says “I apologise” because she feels bad that she made a bad smell and she says “excuse me” because she wants you to accept her apology

A number is equal to the sum of half a second number and 3. The first number is also equal to the sum of one- quarter of the second number and 5. The situation can be represented by using the graph below, where x represents the second number. 16 NO 8 K 3₂ 5 2 4 st 6 8 10 12 14 16 Which equations represent the situation? Hurry ​

Answers

5 1/2
this is how we do it, jus learn abt it

It has been estimated that only about 40% of California residents have adequate earthquake supplies suppose you randomly survey 22 California residents we are interested in the number who have adequate earth quake supplies

Answers

The answers are as follows:

a) X denotes the number of the california residents that have adequate earthquake insurance

b)  x = 1 ,2 ,3 ......

c) P( X=x ) = 0.3(1-0.3) ^ (x-1)

d)  P( X=1) +  P( X=2)  + P( X=3) +  P( X=4)

e) 0.49

f)0.42

g)0.33

What is probability?

It is a branch of mathematics that deals with the occurrence of a random event.

The complete question is

It has been estimated that only about 30% of California residents have adequate earthquake supplies. Suppose we are interested in the number of California residents we must survey until we find a resident who does not have adequate earthquake supplies. a. In words, define the random variable X. b. List the values that Xmay take on. c. Give the distribution of X.X~ _____(_____,_____) d. WhatistheprobabilitythatwemustsurveyjustoneortworesidentsuntilwefindaCaliforniaresidentwhodoes not have adequate earthquake supplies? e. What is the probability that we must survey at least three California residents until we find a California resident who does not have adequate earthquake supplies? f. HowmanyCaliforniaresidentsdoyouexpecttoneedtosurveyuntilyoufindaCaliforniaresidentwhodoesnot have adequate earthquake supplies? g. How many California residents do you expect to need to survey until you find a California resident who does have adequate earthquake supplies?

given that 30% of California residents have adequate earthquake supplies.

a) variable X denotes the number of the california residents that have adequate earthquake insurance

b) x = 1 ,2 ,3 ......

c) p=0.3

P( X=x ) = 0.3(1-0.3) ^ (x-1)

d) P( X=1) +  P( X=2)  + P( X=3) +  P( X=4)

= 0.3(1-0.3) ^ 0 + 0.3(1-0.3) ^1 + 0.3(1-0.3)^ 2 +...

e) P( X≥ 3) = 1- P(X<3)

= 1- (P(X=1) + (X=2))

= 1- 0.051

= 0.49.

f) E(X)= 1/P

p is the resident who does not have adequate earthquake supplies

p= 1-0.3

p = 0.7

E(X) = 1/0.7

= 0.42

g) E(X) =  1/q

= 1/0.3

= 3.333

Learn more about this concept here:

https://brainly.com/question/15512751

#SPJ1

(7 + 7i)(2 − 2i)
(a) Write the trigonometric forms of the complex numbers. (Let
0 ≤ < 2.)

(7 + 7i) =

(2 − 2i) =



(b) Perform the indicated operation using the trigonometric forms. (Let
0 ≤ < 2.)



(c) Perform the indicated operation using the standard forms, and check your result with that of part (b).

Answers

The complex number  -7i into trigonometric form is 7 (cos (90) + sin (90) i) and  3 + 3i in trigonometric form is 4.2426 (cos (45) + sin (45) i)

What is a complex number?

It is defined as the number which can be written as x+iy where x is the real number or real part of the complex number and y is the imaginary part of the complex number and i is the iota which is nothing but a square root of -1.

We have a complex number shown in the picture:

-7i(3 + 3i)

= -7i

In trigonometric form:

z = 7 (cos (90) + sin (90) i)

= 3 + 3i

z = 4.2426 (cos (45) + sin (45) i)

[tex]\rm 7\:\left(cos\:\left(90\right)\:+\:sin\:\left(90\right)\:i\right)4.2426\:\left(cos\:\left(45\right)\:+\:sin\:\left(45\right)\:i\right)[/tex]

[tex]\rm =7\left(\cos \left(\dfrac{\pi }{2}\right)+\sin \left(\dfrac{\pi }{2}\right)i\right)\cdot \:4.2426\left(\cos \left(\dfrac{\pi }{4}\right)+\sin \left(\dfrac{\pi }{4}\right)i\right)[/tex]

[tex]\rm 7\cdot \dfrac{21213}{5000}e^{i\dfrac{\pi }{2}}e^{i\dfrac{\pi }{4}}[/tex]

[tex]\rm =\dfrac{148491\left(-1\right)^{\dfrac{3}{4}}}{5000}[/tex]

=21-21i

After converting into the exponential form:

[tex]\rm =\dfrac{148491\left(-1\right)^{\dfrac{3}{4}}}{5000}[/tex]

From part (b) and part (c) both results are the same.

Thus, the complex number  -7i into trigonometric form is 7 (cos (90) + sin (90) i) and  3 + 3i in trigonometric form is 4.2426 (cos (45) + sin (45) i)

Learn more about the complex number here:

brainly.com/question/10251853

#SPJ1

A hyperbola centered at (7, 0) has a focus at (7, 5) and vertex at (7, 4). Which is the equation of the hyperbola in standard form?

quantity x minus 7 end quantity squared over 16 minus y squared over 9 equals 1
quantity x minus 7 end quantity squared over 25 minus y squared over 16 equals 1
y squared over 16 minus quantity x minus 7 end quantity squared over 9 equals 1
y squared over 25 minus quantity x minus 7 end quantity squared over 16 equals 1

Answers

Based on the calculations, the equation of this hyperbola in standard form is: A. [tex]\frac{x\;-\;7}{16} + \frac{y}{9} = 1[/tex].

How to determine the equation of a hyperbola?

Mathematically, the equation of a hyperbola in standard form is given by:

[tex]\frac{x\;-\;h}{a^2} + \frac{x\;-\;k}{b^2} = 1[/tex]

Given the following data:

Center (h, k) = (7, 0)

Vertex (h+a, k) = (7, 4)

Focus = (h+c, k) = (7, 5)

Also, we can deduce that the value of a and c are 4 and 5 respectively.

For the value of b, we would apply Pythagorean's theorem:

c² = a² + b²

b² = c² - a²

b² = 5² - 4²

b² = 9.

Substituting the parameters into the standard equation, we have:

[tex]\frac{x\;-\;7}{4^2} + \frac{y\;-\;0}{3^2} = 1\\\\\frac{x\;-\;7}{16} + \frac{y}{9} = 1[/tex]

Read more on hyperbola here: https://brainly.com/question/3405939

#SPJ1

Select the correct answer from each drop-down menu.
Consider the function Ax) = 3x+ 1 and the graph of the function g(x) shown below.

Answers

Answer:

write three methods of writing set. Give one example of each methods.

Find an equation for the line that passes through the point P(-5,-3) and is parallel to the line
7x + 4y
10. Use exact values.

Answers

-------------------------------------------------------------------------------------------------------------

Answer:  [tex]\textsf{y = -1.75x - 11.75}[/tex]

-------------------------------------------------------------------------------------------------------------

Given:  [tex]\textsf{Goes through (-5, -3) and parallel to 7x + 4y = 10}[/tex]

Find:  [tex]\textsf{The equation in slope-intercept form}[/tex]

Solution: We need to first solve for y in the equation that was provided so we can determine the slope.  Then we plug in the values into the point-slope form, distribute, simplify, and solve for y to get our final equation.

Subtract 7x from both sides

[tex]\textsf{7x - 7x + 4y = 10 - 7x}[/tex][tex]\textsf{4y = 10 - 7x}[/tex]

Divide both sides by 4

[tex]\textsf{4y/4 = (10 - 7x)/4}[/tex][tex]\textsf{y = (10 - 7x)/4}[/tex][tex]\textsf{y = 10/4 - 7x/4}[/tex][tex]\textsf{y = 2.5 - 1.75x}[/tex]

Plug in the values

[tex]\textsf{y - y}_1\textsf{ = m(x - x}_1\textsf{)}[/tex][tex]\textsf{y - (-3) = -1.75(x - (-5))}[/tex]

Simplify and distribute

[tex]\textsf{y + 3 = -1.75(x + 5)}[/tex][tex]\textsf{y + 3 = (-1.75 * x) + (-1.75 * 5)}[/tex][tex]\textsf{y + 3 = -1.75x - 8.75}[/tex]

Subtract 3 from both sides

[tex]\textsf{y + 3 - 3 = -1.75x - 8.75 - 3}[/tex][tex]\textsf{y = -1.75x - 8.75 - 3}[/tex][tex]\textsf{y = -1.75x - 11.75}[/tex]

Therefore, the final equation in slope-intercept form that follows the information that was provided is y = -1.75x - 11.75

I need help ASAP please

Which statement is true about whether Z and B are independent events?

A: Z and B are independent events because P(Z | B) =P(Z).
B: Z and B are independent events because P(Z | B) = P(B).
C: Z and B are not independent events because P(Z | B) ≠ P(Z).
D: Z and B are not independent events because P(Z | B) ≠ P(B).

Answers

A

Explanation:

Independent event means P(Z|B)=P(Z)

P(Z|B)=(Z ∩ B)/B
=(126)/280
=.45

P(Z)=297/660
=.45

Therefore they are independent events, and the answer is A.
The answer should be Option A

Can someone answer this question for me? Thanks!

Answers

Answer:

The costco basmati rice

Step-by-step explanation:

Under the label it is given that the bag is 18.14 kg

When converted to lbs= 18.14*2.2~39.91

39.91lbs>10lbs, so it has more quantity of rice: Costco Basmati

Answer:

The Costco bag is a better deal as each pound is $0.65 cheaper

Step-by-step explanation:

Let make the weight of each bag in a common unit

18.14kg = about 40 pounds (formula: multiply the mass value by 2.205)

Now find how much it cost per pound for each bag,

Costco: 49.99 / 40 = $1.25 per pound

Bulkmarket: 18.99 / 10 = $1.90 per pound

1.25 < 1.90

four big water bottles can hold 8 gallaons how much can 10 big water gallons hold

Answers

4 can hold 8 gallons = 8/4 = 2 big bottle per gallons.

For 10 big bottles = 10 x 2 = 20 gallons.

If you are looking for how many bottles can be filled with 10 gallons, the answer is 5.
Let x represent the gallons you are finding
If 4=8 then,
10=x
You cross multiply
4x = 10*8
4x /4= 80/4
x = 20

What is the simplified form of √ 144x36?

Answers

Answer:

12x^18

Step-by-step explanation:

Find the area of the trapezoid. TOP 11ft, RIGHT4√3ft , BOTTOM 15ft ,LEFT 8ft

Answers

Answer:

[tex]52\sqrt{3} ft^{2}[/tex]

Step-by-step explanation:

Please refer to the attached picture.

First we will find the area of rectangle BCDE.

Area of Rectangle = Length x Breadth = DE x CD

= 11 x [tex]4\sqrt{3}[/tex]

[tex]=44\sqrt{3} ft^{2}[/tex]

Next we will find Area of Triangle ABE.

Area of Triangle = 0.5 x Base x Height

[tex]0.5*4*4\sqrt{3} \\=8\sqrt{3} ft^{2}[/tex]

Area of Trapezoid = Area of Rectangle + Area of Triangle

[tex]=44\sqrt{3} +8\sqrt{3} \\=52\sqrt{3} ft^{2}[/tex]

Answer:

A = 52[tex]\sqrt{3}[/tex] ft² ≈ 90.1 ft²

Step-by-step explanation:

the area (A) of a trapezoid is calculated as

A = [tex]\frac{1}{2}[/tex] h (b₁ + b₂ )

where h is the perpendicular height and b₁ , b₂ the parallel bases

here h = 4[tex]\sqrt{3}[/tex] , b₁ = 15 , b₂ = 11 , then

A = [tex]\frac{1}{2}[/tex] × 4[tex]\sqrt{3}[/tex] × (15 + 11)

   = 2[tex]\sqrt{3}[/tex] × 26

   = 52[tex]\sqrt{3}[/tex] ft²

   ≈ 90.1 ft² ( to the nearest tenth )

which expression is equivalent to...​

Answers

Answer:

B

Step-by-step explanation:

find the slope of the line that passes through (3,10) and (1,17)​

Answers

Answer:

[tex]m=- \frac{7}{2}[/tex]

Step-by-step explanation:

The slope of a line passing through the two points [tex]\displaystyle{\large{{P}={\left({x}_{{1}},{y}_{{1}}\right)}}}[/tex] and[tex]\displaystyle{\large{{Q}={\left({x}_{{2}},{y}_{{2}}\right)}}}[/tex] is given by [tex]\displaystyle{\large{{m}=\frac{{{y}_{{2}}-{y}_{{1}}}}{{{x}_{{2}}-{x}_{{1}}}}}}[/tex].

We have that [tex]x_1=3[/tex], [tex]y_1=10[/tex], [tex]x_2=1[/tex], [tex]y_2=17[/tex].

Plug the given values into the formula for slope: [tex]m=\frac{\left(17\right)-\left(10\right)}{\left(1\right)-\left(3\right)}=\frac{7}{-2}=- \frac{7}{2}[/tex]

Answer: the slope of the line is [tex]m=- \frac{7}{2}[/tex].

Answer:

slope = - [tex]\frac{7}{2}[/tex]

Step-by-step explanation:

calculate the slope m using the slope formula

m = [tex]\frac{x_{2}-y_{1} }{x_{2}-x_{1} }[/tex]

with (x₁, y₁ ) = (3, 10 ) and (x₂, y₂ ) = (1, 17 )

m = [tex]\frac{17-10}{1-3}[/tex] = [tex]\frac{7}{-2}[/tex] = - [tex]\frac{7}{2}[/tex]

What are the solutions to the equations y= 4x^2+5x-6

Answers

Answer:

D. x = -2; x = 3/4

Step-by-step explanation:

Read the values of a, b, and c from the quadratic equation: a is the number in front of x^2, b is the number in front of x, c is the number at the end. In our case: a = 4,b = 5,c = −6 The formula for the roots is =  [tex]\frac{-b +-\sqrt{b^{2}-4ac } }{2a\\}[/tex]

March 8, 2017, one U.S. dollar was worth 66.79 Indian rupees.
a) On that date, how many dollars was 110.66 rupees worth?
Round your answer to the nearest hundredth of a dollar. I need help with this question.

Answers

[tex] \huge \tt \underline {\green{Answer}}[/tex]

If on March 8, 2017 , one U.S. dollar worth 66.79 Indian rupees

ie. $1 = Rs 66.79

$ 1 = 66.79 × 1

$ ? = 110.66

$ = New / old

$ = 110.66 / 66.79

$ = 1.65683485552

or

$1.66 = 110.66

How many pounds of candy that sells for ​$0.82 per lb must be mixed with candy that sells for ​$1.36 per lb to obtain 9 lb of a mixture that should sell for ​$0.91 per​ lb?

Answers

7.5 pounds of the $0.82 per lb candy must be used in the mixture.

How many pounds of each candy should we use?

First, let's define the variables:

x = pounds of the $0.82 candy used.y = pounds of the $1.36 candy used.

We want to make 9 lb of mixture, then:

x + y = 9.

And the price of these 9 pounds must be $0.91, then we can write:

x*$0.82 + y*$1.36 = 9*$0.91 = $8.19

Then we have a system of equations:

x + y = 9.

x*$0.82 + y*$1.36 = $8.19

We can isolate y on the first equation so we get:

y = 9 - x

Now we can replace that on the other equation:

x*$0.82 + (9 - x)*$1.36 = $8.19

And now we can solve this for x.

x*($0.82 - $1.36) = $8.19 - 9*$1.36

-x*$0.54 = -$4.05

x = (4.05/0.54) = 7.5

So 7.5 pounds of the $0.82 per lb candy must be used in the mixture.

If you want to learn more about systems of equations:

https://brainly.com/question/13729904

#SPJ1

can someone help me asap :)

Answers

Answer:

Option 2 - TU is perpendicular to MN and MN is parallel to PQ

angle M = angle P ( 90°)

so MN is parallel to pq

MN makes 90° with UT so it is perpendicular

Other Questions
Helpp me answer this question for 10 points part2 someone help how do I factor x^3-y^3 solve the compound inequality -5 need help asap willing to give a lot of points! What is the change in freezingpoint, ATf, when the 0.195 maqueous solution of K2S dissolvesinto 3 particles? _____ is the art and science of trying to get people to do what you want (dok 1, standard standard 2.1.n evaluate internal and external influences that affect food choices.) testimonial salience persuasion repetition Select ALL the correct answers.Which are the three main reasons Kennedy gives to develop his argument?No countries except the United States are even attempting to explore space, and someone must do it.Only the United States is equipped to ensure that knowledge gained from space exploration is used for good.Recent advances in technology have brought a Moon landing within US capabilities.The United States is currently behind in the effort to get to the Moon, and this must be changed.Launching a rocket to the Moon is a similar challenge to climbing a high mountain or sailing across the ocean.The rockets and satellites of the United States are far better than those of other countries. which of the following choices is an example of drug abuse1. taking a Tylenol every day for a week for a headache 2. using a friend's prescription cough syrup for your own cough 3. smoking an illegal drug on a regular basis 4. all of these choices are correct Answer the questions using 1 complete Spanish sentence.Qu pasar con la capa de ozono si seguimos utilizando aerosoles?Crees que el hombre dejar de contaminar el medio ambiente?Opinas que la humanidad se preocupar por el calentamiento global cuando vea ms desastres naturales?Me recomiendas que recicle para que haya menos contaminacin?Por qu no es bueno que dejemos basura en el campo? 30 POINTS FOR THE CORRECT ANSWERSThinking globally has become a big part of visual communications especially with online communication. Images, words, and designs can instantly travel the world.Questions to Consider and Answer:How does a designer decide what images, colors, and styles will appeal to a global audience?Is it possible to design something that appeals to everyone?What are the biggest potential problems with global design?Are there legal implications to consider?What is the best way to understand another culture?Put yourself in the shoes of a designer who has been asked to create an ad campaign for a global athletic footwear company. Give a brief description of how you would plan to do research on different cultures and what potential obstacles and opportunities you might have. Sketch an ad that could appear on a website anywhere in the world. It must communicate simply and not be offensive to any culture or group of people. Post your sketch with your answers to the questions above. In "Water Flowing from Toilet to Tap May Be Hard to Swallow, what reasons does Schwartz give to support his claim? Select two options. A passenger is running inside a stationary train with some speed and looks at the person standing on the platform. Describe the motion of the person standing on the platform with respect to the running passenger. If you take a 10.0 mL portion of a 14.8 M solution of NH3 and dilute it to a total solution volume of 0.500 L, what will be the concentration of the final solution To what extent do you agree that the UK should provide source of food locally rather than importing food from abroad Read the excerpt fromJulius Caesar, act 1, scene 2This soothsayer's warning in this passage is an exampleSOOTHSAYER. Beware the ides of March.CAESAR. What man is that?figurative language.BRUTUS. A soothsayer bids you beware the ides of March.CAESAR. Set him before me; let me see his face.CASSIUS. Fellow, come from the throng; look upon Caesar.CAESAR. What say'st thou to me now? Speak once again.SOOTHSAYER. Beware the ides of March.The smoothsayers warning in this passage is an example of:a) figurative language b) inversions c) foreshadowing d) hyperbole This year you have decided to focus your efforts on marketing to employer group plans. One employer provides you with a list of their retirees and asks you to contact them to explain the characteristics of the plan they have selected. What should you do?. marshall corporation had $220,000 in invested assets, sales of: How were citizens of Rome involved in the political process?O Roman citizens had no input on ancient Rome's political process but were involved in electing public officials.O Roman citizens eamed voting rights from the Senate, who decided which citizens could voteO Roman citizens could vote on the Senate's legislation, and some citizens were involved in electing public officials.O Roman citizens only participated by serving in the military IB Chemistry SL Question - Empirical/Molecular formulaeA 4.406g sample of a compound containing only C, H and O was burnt in excess oxygen. 8.802g of CO2 and 3.604g of H2O were produced.i) Determine the empirical formula of the compound (section 6 of the data booklet)ii) Determine the molecular formula of this compound if its molar mass is 88.12gmol-1 Which of these areas is an example of a consolidated metropolitan statistical area?A the Pacific NorthwestB.Los Angeles-Long Beach-AnaheimC New York City, not Including its suburbsD. any two cities with similar demographics